Ch. 49,

¡Supera tus tareas y exámenes ahora con Quizwiz!

Which information will the nurse include in teaching a female patient who has peripheral arterial disease, type 2 diabetes, and sensory neuropathy of the feet and legs? a. Choose flat-soled leather shoes. b. Set heating pads on a low temperature. c. Use callus remover for corns or calluses. d. Soak feet in warm water for an hour each day.

ANS: A The patient is taught to avoid high heels and that leather shoes are preferred. The feet should be washed, but not soaked, in warm water daily. Heating pad use should be avoided. Commercial callus and corn removers should be avoided. The patient should see a specialist to treat these problems

After change-of-shift report, which patient should the nurse assess first? a. 19-year-old with type 1 diabetes who has a hemoglobin A1C of 12% b. 23-year-old with type 1 diabetes who has a blood glucose of 40 mg/dL c. 40-year-old who is pregnant and whose oral glucose tolerance test is 202 mg/dL d. 50-year-old who uses exenatide (Byetta) and is complaining of acute abdominal pain

ANS: B Because the brain requires glucose to function, untreated hypoglycemia can cause unconsciousness, seizures, and death. The nurse will rapidly assess and treat the patient with low blood glucose. The other patients also have symptoms that require assessments and/or interventions, but they are not at immediate risk for life-threatening complications

A 51-year-old patient with diabetes mellitus is scheduled for a fasting blood glucose level at 8:00 AM. The nurse instructs the patient to only drink water after what time? A) 6:00 PM on the evening before the test B) Midnight before the test C) 4:00 AM on the day of the test D)7:00 AM on the day of the test

B) Midnight before the test Typically, a patient is ordered to be NPO for 8 hours before a fasting blood glucose level. For this reason, the patient who has a lab draw at 8:00 AM should not have any food or beverages containing any calories after midnight.

A nurse ho is teaching a client how to recognize symptoms of hypoglycemia should include which symptoms in the teaching? (Select all that apply.) a. Headache b. Nervousness c. Bradycardia d. Sweating e. Thirst f. Sweet breath odor

a. Headache b. Nervousness d. Sweating

In type 1 diabetes there is an osmotic effect of glucose when insulin deficiency prevents the use of glucose for energy. Which classic symptom is caused by the osmotic effect of glucose? a. fatigue b. polydipsia c. polyphagia d. recurrent infections

b. polydipsia

A client is diagnosed with type 2 diabetes mellitus. The nurse is aware that which statement is true? a. Client is most likely a teenager. b. Client is most likely a child younger than 10 years. c. Heredity is a major causative factor. d. Viral infections contribute most to disease development.

c. Heredity is a major causative factor.

the nurse is discussing the importance of exercising with a client diagnosed with type 2 diabetes who diabetes is well controlled with diet and exercise. Which information should the nurse include in the teaching about diabetes? a. eat a simple carb snack before exercising b. carry peanut butter crackers when exercising c. encourage the client to walk 20 minutes three times a week d. perform warm up and cool down exercises

d. perform warm up and cool down exercises

What is the rationale for rotation of insulin injection sites? a. prevents polyuria b. prevents rejection of insulin c. prevents an allergic reaction d. prevents lipodystrophy

d. prevents lipodystrophy

A diabetic patient is admitted with ketoacidosis and the health care provider writes all of the following orders. Which order should the nurse implement first? a. Start an infusion of regular insulin at 50 U/hr. b. Give sodium bicarbonate 50 mEq IV push. c. Infuse 1 liter of normal saline per hour. d. Administer regular IV insulin 30 U.

C Rationale: The most urgent patient problem is the hypovolemia associated with DKA, and the priority is to infuse IV fluids. The other actions can be accomplished after the infusion of normal saline is initiated. Cognitive Level: Application Text Reference: p. 1280 Nursing Process: Implementation NCLEX: Physiological Integrity

Which assessment data indicate the client diagnosed with DKA is responding to the medical treatment? a. the client has tended skin turgor and dry mucous membranes b. the client is alert and oriented times three c. the clients ABG results are pH 7.29, PaCO2 44, HCO3 15 d. the clients serum potassium level is 3.3 mEq/L

b. the client is alert and oriented times three

A 32-year-old patient with diabetes is starting on intensive insulin therapy. Which type of insulin will the nurse discuss using for mealtime coverage? a. Lispro (Humalog) b. Glargine (Lantus) c. Detemir (Levemir) d. NPH (Humulin N)

ANS: A Rapid- or short-acting insulin is used for mealtime coverage for patients receiving intensive insulin therapy. NPH, glargine, or detemir will be used as the basal insulin.

A 34-year-old has a new diagnosis of type 2 diabetes. The nurse will discuss the need to schedule a dilated eye exam a. every 2 years. b. as soon as possible. c. when the patient is 39 years old. d. within the first year after diagnosis.

ANS: B Because many patients have some diabetic retinopathy when they are first diagnosed with type 2 diabetes, a dilated eye exam is recommended at the time of diagnosis and annually thereafter. Patients with type 1 diabetes should have dilated eye exams starting 5 years after they are diagnosed and then annually.

Which statement by the patient with type 2 diabetes is accurate? a. "I am supposed to have a meal or snack if I drink alcohol." b. "I am not allowed to eat any sweets because of my diabetes." c. "I do not need to watch what I eat because my diabetes is not the bad kind." d. "The amount of fat in my diet is not important; it is just the carbohydrates that raise my blood sugar."

a. "I am supposed to have a meal or snack if I drink alcohol."

In nutritional management of all types of diabetes, it is important for the patient to a. eat regular meals at regular times. b. restrict calories to promote moderate weight loss. c. eliminate sucrose and other simple sugars from the diet. d. limit saturated fat intake to 30% of dietary intake.

a. Rationale: The body requires food at regularly spaced intervals throughout the day, and omission or delay of meals can result in hypoglycemia, especially for the patient taking insulin or oral hypoglycemic agents. Weight loss may be recommended in type 2 diabetes if the individual is overweight, but many patients with type 1 diabetes are thin and require an increase in caloric intake. Fewer than 7% of total calories should be from saturated fats, and simple sugars should be limited, but moderate amounts can used if counted as a part of total carbohydrate intake.

individualized nutrition therapy for patients using conventional, fixed insulin regimens should include teaching the patient to a. eat regular meals at regular times b. restrict calories to promote moderate weight loss c. eliminate sucrose and other simple sugars from the diet d. limit saturated fat intake to 30% of dietary calorie intake

a. eat regular meals at regular times

the patient with diabetes has a blood glucose level of 248 mg/dL. Which manifestations in the patient would the nurse understand as being related to this blood glucose level? (select all that apply) a. headache b. unsteady gait c. abdominal cramps d. emotional changes e. increase in urination f. weakness and fatigue

a. headache c. abdominal cramps e. increase in urination f. weakness and fatigue

The elderly client is admitted to the intensive care department diagnosed with severe HHNS. which collaborative intervention should the nurse include in the plan of care? a. infuse 0.9% normal saline IV b. administer intermediate-acting insulin c. perform a blood glucometer cheeks daily d. monitor arterial blood gas results

a. infuse 0.9% normal saline IV

What are the major symptoms that characterize diabetes? (Select all that apply.) a. polyuria b. polyphagia c. polyposia d. polydipsia e. polyrrhea

a. polyuria b. polyphagia d. polydipsia

What are manifestations of DKA (select all that apply) a. thirst b. ketonuria c. dehydration d. metabolic acidosis e. Kussmaul respirations f. sweet, fruity breath odor

a. thirst b. ketonuria c. dehydration d. metabolic acidosis e. Kussmaul respirations f. sweet, fruity breath odor

Analyze the following diagnostic findings for your patient with type 2 diabetes. Which result will need further assessment? a. BP 126/80 mm Hg b. A1C 9% c. FBG 130mg/dL (7.2 mmol/L) d. LDL cholesterol 100 mg/dL (2.6 mmol/L)

b. A1C 9%

The nurse at a freestanding health care clinic is caring for a 56 year old male client who is homeless and is a type 2 diabetic controlled with insulin. Which action is an example of client advocacy? a. ask the client if he has somewhere he can go and live b. arrange for someone to give him insulin at a local homeless shelter c. notify adult protective services about the clients situation d. ask the HCP to take the client off insulin because he is homeless

b. arrange for someone to give him insulin at a local homeless shelter

What should the goals of nutrition therapy for the patient with type two diabetes include? a. ideal body weight b. normal serum glucose and lipid levels c. a special diabetic diet using dietetic foods d. five small meals per day with a bedtime snack

b. normal serum glucose and lipid levels

A patient with Type 1 diabetes uses 20 U of 70/30 neutral protamine Hagedorn (NPH/regular) in the morning and at 6 pm. When teaching the patient about this regimen, what should the nurse emphasize? a. hypoglycemia is most likely to occur before the noon meal b. flexibility in food intake is possible because insulin is available 24 hours a day c. a set meal pattern with a bedtime snack is necessary to prevent hypoglycemia d. premeal glucose checks are required to determine needed changes in daily dosing

c. a set meal pattern with a bedtime snack is necessary to prevent hypoglycemia

A patient taking insulin has recorded fasting glucose levels above 200 mg/dL (11.1 mmol/L) on awakening for the last five mornings. What should the nurse advise the patient to do first? a. increase the evening insulin dose to prevent the dawn phenomenon b. use a single-dose insulin regimen with an intermediate acting insuling c. monitor the glucose level at bedtime, between 2 AM and 4 AM, and on arising d. decrease the evening insulin dosage to prevent night hypoglycemia and the Somogyi effect

c. monitor the glucose level at bedtime, between 2 AM and 4 AM, and on arising

A diabetic patient has a serum glucose level of 824 mg/dL (45.7 mmol/L) and is unresponsive. Following assessment of the patient, the nurse suspects diabetic ketoacidosis rather than hyperosmolar hyperglycemic syndrome based on the findings of a. polyuria. b. severe dehydration. c. rapid, deep respirations. d. decreased serum potassium.

c. rapid, deep respirations.

The nurse is assessing the feet of a client with long term type 2 diabetes. Which assessment data warrant immediate intervention by the nurse? a. the client has crumbling toe nails b. the client has athletes foot c. the client has a necrotic big toe d.the client has thickened toenails

c. the client has a necrotic big toe

Which statement would be correct for a patient with type 2 diabetes who was admitted to the hospital with pneumonia? a.The patient must receive insulin therapy to prevent ketoacidosis. b.The patient has islet cell antibodies that have destroyed the pancreas's ability to produce insulin. c.The patient has minimal or absent endogenous insulin secretion and requires daily insulin injections. d.The patient may have sufficient endogenous insulin to prevent ketosis but is at risk for hyperosmolar hyperglycemic syndrome.

d.The patient may have sufficient endogenous insulin to prevent ketosis but is at risk for hyperosmolar hyperglycemic syndrome.

The nurse is teaching the patient with prediabetes ways to prevent or delay the development of type two diabetes. What information should be included (select all that apply). a. maintain a healthy weight b. exercise for 60 minutes each day c. have BP checked regularly d. assess for visual changes on monthly basis e. monitor for polyuria, polyphagia, and polydipsia

a. maintain a healthy weight e. monitor for polyuria, polyphagia, and polydipsia

Which class of oral glucose-lowering agents is most commonly used for people with type 2 diabetes because is reduces hepatic glucose production and enhances tissue uptake of glucose? a. insulin b. biguanide c. meglitinide d. sulfonylurea

b. biguanide

what disorders and diseases are related to macrovascular complications of diabetes? select all that apply a. chronic kidney disease b. coronary artery disease c. microaneurysms and destruction or retinal vessels d. ulceration and amputation of the lower extremities e. capillary and arteriole membrane thickening specific to diabetes

b. coronary artery disease d. ulceration and amputation of the lower extremities

A client is receiving a daily dose of Humulin N insulin at 7:30 am. The nurse expects the peak effect of this drug to occur at which time? a. 8:15 am b. 10:30 am c. 5:00 pm d. 11:00 pm

c. 5:00 pm

What is the only type of insulin that may be administered IV? a. NPH b. Detemir c. Lantus d. Regular

d. Regular

Which information should be included in health teaching for clients taking insulin? (Select all that apply.) a. Recognize signs of hypoglycemic reaction. b. Adhere to the prescribed diet. c. Take insulin as prescribed. d. Monitor blood glucose level. e. Be sure to exercise. f. Keep appointments with health care provider. g. Alter insulin dose based on how you're feeling.

a. Recognize signs of hypoglycemic reaction. b. Adhere to the prescribed diet. c. Take insulin as prescribed. d. Monitor blood glucose level.

Which clinical manifestations may be seen in a client experiencing a hypoglycemic (insulin) reaction? (Select all that apply.) a. headache b. nervousness c. tremor d. excessive perspiration e. tachycardia f. abdominal pain

a. headache b. nervousness c. tremor d. excessive perspiration e. tachycardia

the client is admitted to the ICU diagnosed with DKA. Which intervention should the nurse implement? select all that apply a. maintain adequate ventilation b. assess fluid volume status c. administer IV potassium d. check for urinary ketones e. monitor intake and output

a. maintain adequate ventilation b. assess fluid volume status c. administer IV potassium d. check for urinary ketones e. monitor intake and output

Which electrolyte replacement should the nurse anticipate being ordered by the HCP in the client diagnosed with DKA who has been admitted to the ICU? a. glucose b. potassium c. calcium d. sodium

b. potassium

A 27-year-old patient admitted with diabetic ketoacidosis (DKA) has a serum glucose level of 732 mg/dL and serum potassium level of 3.1 mEq/L. Which action prescribed by the health care provider should the nurse take first? a. Place the patient on a cardiac monitor. b. Administer IV potassium supplements. c. Obtain urine glucose and ketone levels. d. Start an insulin infusion at 0.1 units/kg/hr.

ANS: A Hypokalemia can lead to potentially fatal dysrhythmias such as ventricular tachycardia and ventricular fibrillation, which would be detected with electrocardiogram (ECG) monitoring. Because potassium must be infused over at least 1 hour, the nurse should initiate cardiac monitoring before infusion of potassium. Insulin should not be administered without cardiac monitoring because insulin infusion will further decrease potassium levels. Urine glucose and ketone levels are not urgently needed to manage the patient's care.

After the home health nurse has taught a patient and family about how to use glargine and regular insulin safely, which action by the patient indicates that the teaching has been successful? a. The patient disposes of the open insulin vials after 4 weeks. b. The patient draws up the regular insulin in the syringe and then draws up the glargine. c. The patient stores extra vials of both types of insulin in the freezer until needed. d. The patient's family prefills the syringes weekly and stores them in the refrigerator.

A Rationale: Insulin can be stored at room temperature for 4 weeks. Glargine should not be mixed with other insulins or prefilled and stored. Freezing alters the insulin molecule and should not be done. Cognitive Level: Application Text Reference: p. 1261 Nursing Process: Evaluation NCLEX: Physiological Integrity

To monitor for complications in a patient with type 2 diabetes, which tests will the nurse in the diabetic clinic schedule at least annually (select all that apply)? a. Chest x-ray b. Blood pressure c. Serum creatinine d. Urine for microalbuminuria e. Complete blood count (CBC) f. Monofilament testing of the foot

ANS: B, C, D, F Blood pressure, serum creatinine, urine testing for microalbuminuria, and monofilament testing of the foot are recommended at least annually to screen for possible microvascular and macrovascular complications of diabetes. Chest x-ray and CBC might be ordered if the diabetic patient presents with symptoms of respiratory or infectious problems but are not routinely included in screening

Which of these laboratory values noted by the nurse when reviewing the chart of a diabetic patient indicates the need for further assessment of the patient? a. Fasting blood glucose of 130 mg/dl b. Noon blood glucose of 52 mg/dl c. Glycosylated hemoglobin of 6.9% d. Hemoglobin A1C of 5.8%

B Rationale: The nurse should assess the patient with a blood glucose level of 52 mg/dl for symptoms of hypoglycemia, and give the patient some carbohydrate-containing beverage such as orange juice. The other values are within an acceptable range for a diabetic patient. Cognitive Level: Application Text Reference: pp. 1281-1282 Nursing Process: Assessment NCLEX: Physiological Integrity

A patient with type 2 diabetes has sensory neuropathy of the feet and legs and peripheral vascular disease evidenced by decreased peripheral pulses and dependent rubor. The nurse teaches the patient that a. the feet should be soaked in warm water on a daily basis. b. flat-soled leather shoes are the best choice to protect the feet from injury. c. heating pads should always be set at a very low temperature. d. over-the-counter (OTC) callus remover may be used to remove callus and prevent pressure.

B Rationale: The patient is taught to avoid high heels and that leather shoes are preferred. The feet should be washed, but not soaked, in warm water daily. Heating pad use should be avoided. Commercial callus and corn removers should be avoided; the patient should see a specialist to treat these problems. Cognitive Level: Application Text Reference: p. 1287 Nursing Process: Implementation NCLEX: Health Promotion and Maintenance

A patient who has just been diagnosed with type 2 diabetes is 5 ft 4 in (160 cm) tall and weighs 182 pounds (82 kg). A nursing diagnosis of imbalanced nutrition: more than body requirements is developed. Which patient outcome is most important for this patient? a. The patient will have a diet and exercise plan that results in weight loss. b. The patient will state the reasons for eliminating simple sugars in the diet. c. The patient will have a glycosylated hemoglobin level of less than 7%. d. The patient will choose a diet that distributes calories throughout the day.

C Rationale: The complications of diabetes are related to elevated blood glucose, and the most important patient outcome is the reduction of glucose to near-normal levels. The other outcomes are also appropriate but are not as high in priority. Cognitive Level: Application Text Reference: p. 1273 Nursing Process: Planning NCLEX: Physiological Integrity

The nurse is teaching a patient with type 2 diabetes mellitus about exercise to help control his blood glucose. The nurse knows the patient understands when the patient elicits which exercise plan? A) "I want to go fishing for 30 minutes each day; I will drink fluids and wear sunscreen." B) "I will go running each day when my blood sugar is too high to bring it back to normal." C) "I will plan to keep my job as a teacher because I get a lot of exercise every school day." D)"I will take a brisk 30-minute walk 5 days per week and do resistance training 3 times a week."

D)"I will take a brisk 30-minute walk 5 days per week and do resistance training 3 times a week." The best exercise plan for the person with type 2 diabetes is for 30 minutes of moderate activity 5 days per week and resistance training 3 times a week. Brisk walking is moderate activity. Fishing and teaching are light activity, and running is considered vigorous activity.

The nurse is caring for a patient after a parathyroidectomy. The nurse would prepare to administer IV calcium gluconate if the patient exhibits which clinical manifestations?

Facial muscle spasms and laryngospasms (manifestations of hypocalcemia)

Antidiabetic drugs are designed to control signs and symptoms of diabetes mellitus. The nurse primarily expects a decrease in which? a. Blood glucose b. Fat metabolism c. Glycogen storage d. Protein mobilization

a. Blood glucose

A patient with diabetes calls the clinic because she is experiencing nausea and flu like symptoms. Which advice from the nurse will be the best for this patient? a. administer the usual insulin dosage b. hold fluid intake until the nausea subsides c. come to the clinic immediately for evaluation and treatment d. monitor the blood glucose everyone one to two hours and call if it rises over 150 mg/dL (8.3 mmol/L)

a. administer the usual insulin dosage

the home health nurse is completing the admission assessment for a 76 year old client diagnosed with type 2 diabetes controlled with 70/30 insulin. Which intervention should be included in the plan of care a. assess the clients ability to read small print b. monitor the clients serum pt level c. teach the client how to perform a hemoglobin A1C test daily d. instruct the client to check the feet weekly

a. assess the clients ability to read small print

A nurse working in an outpatient clinic plans a screening program for diabetes. Recommendations for screening would include a. OGTT fro all minority populations every year. b. FPG for all individuals at age 45 and then every 3 years. c. testing all people under the age of 21 for islet cell antibodies. d. testing for type 2 diabetes only in overweight or obese individuals.

b. Rationale: The American Diabetes Association recommends that testing for type 2 diabetes with a FPG should be considered for all individuals at the age of 45 and above and, if normal, repeated every 3 years. Testing for immune markers of type 1 diabetes is not recommended. Testing of a younger age or more frequently should be done for members of a high-risk ethnic population, including African Americans, Hispanics, Native Americans, Asians Americans, and Pacific Islanders.

In diabets, atherosclerotic disease affecting the cerebrovascular, cardiovascular, and peripheral vascular systems a. can be prevent by tight glucose control. b. occurs with a higher frequency and earlier onset than in the nondiabetic population. c. is cause by the hyperinsulinemia related to insulin resistance common in type 2 diabetes. d. cannot be modified by reduction of risk factors such as smoking, obesity, and high fat intake.

b. Rationale: The development of atherosclerotic vessel disease seems to be promoted by the altered lipid metabolism common to diabetes, and although tight glucose control may help delay the process, it does not prevent it completely. Atherosclerosis in diabetic patients does respond somewhat to a reduction in general risk factors, as it does in nondiabetics, and reduction in fat intake, control of hypertension, abstention from smoking, maintenance of normal weight, and regular exercise should be carried out by diabetic patients.

an 18 year old female client, 5'4" tall, weighting 113 kg, comes to the clinic for a nonhealing wound on her lower leg, which she has had for two weeks. Which disease process should the nurse suspect the client has developed? a. Type 1 diabetes b. Type 2 diabetes c. gestational diabetes d. acanthosis nigricans

b. Type 2 diabetes

The pt with diabetes has been diagnosed with autnomic neurupathy. What problems should the nurse expect to find in the patient (select all that apply) a. painless foot ulcers b. erectile dysfunction c. burning foot pain at night d. loss of fine motor control e. vomiting undigested food f. painless myocardial infarction

b. erectile dysfunction e. vomiting undigested food f. painless myocardial infarction

A client is to receive insulin before breakfast, and the time of breakfast tray delivery is variable. The nurse knows that which insulin should not be administered until the breakfast tray has arrived and the client is ready to eat? a. Humulin N b. lispro (Humalog) c. glargine (Lantus) d. Humulin R

b. lispro (Humalog)

The patient with newly diagnosed diabetes is displaying shakiness, confusion, irritability, and slurred speech. What should the nurse suspect is happening a. DKA b. HHS c. hypoglycemia d. hyperglycemia

c. hypoglycemia

The charge nurse is making client assignments in the ICU. Which client should be assigned to the most experienced nurse? a. the client with type 2 diabetes who has a blood glucose level of 348 mg/dL b. the client diagnosed with type 1 diabetes who is experiencing hypoglycemia c. the client with DKA who has multifocal premature ventricular contractions d. the client with HHNS who has a plasma osmolarity of 290 mOsm/L

c. the client with DKA who has multifocal premature ventricular contractions

The client diagnosed with Type 1 diabetes has a glycosylated hemoglobin (A1C) of 8.1%. which interpretation should the nurse make based on this result? a. this result is below normal levels b. this result is within acceptable levels c. this result is above recommended levels d. this result is dangerously high

c. this result is above recommended levels

Following the teaching of foot care to a diabetic patient, the nurse determines that additional instruction is needed when the patient says, a. "I should wash my feet daily with soap and warm water." b. "I should always wear shoes to protect my feet from injury." c. "If my feet are cold, I should wear socks instead of using a heating pad." d. "I'll know if I have sores or lesions on my feet because they will be painful."

d. Rationale: Complete or partial loss of sensitivity of the feet is common with peripheral neuropathy of diabetes, and diabetics may suffer foot injury and ulceration without ever having pain. Feet must be inspected during daily care for any cuts, blisters, swelling, or reddened areas.

Lispro insulin (Humalog) with NPH insulin is ordered for a patient with newly diagnosed type 1 diabetes. The nurse knows that when lispro insulin is used, it should be administered a. only once a day b. 1 hour before meals c. 30-45 minutes before meals d. at mealtime or within 15 minutes of meals

d. Rationale: Lispro is a rapid-acting insulin that has an onset of action of 5 to 15 minutes and should be injected at the time of the meal to within 15 minutes of eating. Regular insulin is short acting with an onset of action in 30 to 60 minutes following administration and should be given 30 to 45 minutes before meals.

In addition to promoting the transport of glucose from the blood into the cell, what does insulin do? a. enhances the breakdown of adipose tissue for energy b. stimulates hepatic glycogenolysis and gluconeogenesis c. prevents the transport of triglycerides into adipose tissue d. accelerates the transport of amino acids into cells and their synthesis into protein

d. accelerates the transport of amino acids into cells and their synthesis into protein

Intramuscular glucagon is administered to an unresponsive patient for treatment of hypoglycemia. Which action should the nurse take after the patient regains consciousness? a. Give the patient a snack of cheese and crackers. b. Have the patient drink a glass of orange juice or nonfat milk. c. Administer a continuous infusion of 5% dextrose for 24 hours. d. Assess the patient for symptoms of hyperglycemia.

A Rationale: Rebound hypoglycemia can occur after glucagon administration, but having a meal containing complex carbohydrates plus protein and fat will help prevent hypoglycemia. Orange juice and nonfat milk will elevate blood sugar rapidly, but the cheese and crackers will stabilize blood sugar. Administration of glucose intravenously might be used in patients who were unable to take in nutrition orally. The patient should be assessed for symptoms of hypoglycemia after glucagon administration. Cognitive Level: Application Text Reference: p. 1282 Nursing Process: Implementation NCLEX: Physiological Integrity

The nurse has been teaching the patient to administer a dose of 10 units of regular insulin and 28 units of NPH insulin. The statement by the patient that indicates a need for additional instruction is, a. "I need to rotate injection sites among my arms, legs, and abdomen each day." b. "I will buy the 0.5-ml syringes because the line markings will be easier to see." c. "I should draw up the regular insulin first after injecting air into the NPH bottle." d. "I do not need to aspirate the plunger to check for blood before I inject the insulin."

A Rationale: Rotating sites is no longer necessary because all insulin is now purified human insulin, and the risk for lipodystrophy is low. The other patient statements are accurate and indicate that no additional instruction is needed. Cognitive Level: Application Text Reference: p. 1262 Nursing Process: Evaluation NCLEX: Health Promotion and Maintenance

Which statement by a nurse to a patient newly diagnosed with type 2 diabetes is correct? a. Insulin is not used to control blood glucose in patients with type 2 diabetes. b. Complications of type 2 diabetes are less serious than those of type 1 diabetes. c. Changes in diet and exercise may control blood glucose levels in type 2 diabetes. d. Type 2 diabetes is usually diagnosed when the patient is admitted with a hyperglycemic coma.

ANS: C For some patients with type 2 diabetes, changes in lifestyle are sufficient to achieve blood glucose control. Insulin is frequently used for type 2 diabetes, complications are equally severe as for type 1 diabetes, and type 2 diabetes is usually diagnosed with routine laboratory testing or after a patient develops complications such as frequent yeast infections.

Cardiac monitoring is initiated for a patient in diabetic ketoacidosis (DKA). The nurse recognizes that this measure is important to identify a. electrocardiographic (ECG) changes and dysrhythmias related to hypokalemia. b. fluid overload resulting from aggressive fluid replacement. c. the presence of hypovolemic shock related to osmotic diuresis. d. cardiovascular collapse resulting from the effects of hyperglycemia.

A Rationale: The hypokalemia associated with metabolic acidosis can lead to potentially fatal dysrhythmias such as ventricular tachycardia and ventricular fibrillation, which would be detected with ECG monitoring. Fluid overload, hypovolemia, and cardiovascular collapse are possible complications of DKA, but cardiac monitoring would not detect theses. Cognitive Level: Application Text Reference: p. 1281 Nursing Process: Assessment NCLEX: Physiological Integrity

While hospitalized and recovering from an episode of diabetic ketoacidosis, the patient calls the nurse and reports feeling anxious, nervous, and sweaty. Based on the patient's report, the nurse should a. obtain a glucose reading using a finger stick. b. administer 1 mg glucagon subcutaneously. c. have the patient eat a candy bar. d. have the patient drink 4 ounces of orange juice.

A Rationale: The patient's clinical manifestations are consistent with hypoglycemia and the initial action should be to check the patient's glucose with a finger stick or order a stat blood glucose. If the glucose is low, the patient should ingest a rapid-acting carbohydrate, such as orange juice. Glucagon might be given if the patient's symptoms become worse or if the patient is unconscious. Candy bars contain fat, which would slow down the absorption of sugar and delay the response to treatment. Cognitive Level: Application Text Reference: p. 1282 Nursing Process: Implementation NCLEX: Physiological Integrity

When assessing the patient experiencing the onset of symptoms of type 1 diabetes, which question should the nurse ask? a. "Have you lost any weight lately?" b. "Do you crave fluids containing sugar?" c. "How long have you felt anorexic?" d. "Is your urine unusually dark-colored?"

A Rationale: Weight loss occurs because the body is no longer able to absorb glucose and starts to break down protein and fat for energy. The patient is thirsty but does not necessarily crave sugar- containing fluids. Increased appetite is a classic symptom of type 1 diabetes. With the classic symptom of polyuria, urine will be very dilute. Cognitive Level: Application Text Reference: pp. 1255, 1258 Nursing Process: Assessment NCLEX: Physiological Integrity

The nurse is assisting a patient with newly diagnosed type 2 diabetes to learn dietary planning as part of the initial management of diabetes. The nurse would encourage the patient to limit intake of which foods to help reduce the percent of fat in the diet? A) Cheese B) Broccoli C) Chicken D) Oranges

A) Cheese Cheese is a product derived from animal sources and is higher in fat and calories than vegetables, fruit, and poultry. Excess fat in the diet is limited to help avoid macrovascular changes.

The nurse has been teaching a patient with diabetes mellitus how to perform self-monitoring of blood glucose (SMBG). During evaluation of the patient's technique, the nurse identifies a need for additional teaching when the patient does what? A) Chooses a puncture site in the center of the finger pad. B) Washes hands with soap and water to cleanse the site to be used. C) Warms the finger before puncturing the finger to obtain a drop of blood. D) Tells the nurse that the result of 110 mg/dL indicates good control of diabetes.

A) Chooses a puncture site in the center of the finger pad. The patient should select a site on the sides of the fingertips, not on the center of the finger pad as this area contains many nerve endings and would be unnecessarily painful. Washing hands, warming the finger, and knowing the results that indicate good control all show understanding of the teaching.

The nurse is evaluating a 45-year-old patient diagnosed with type 2 diabetes mellitus. Which symptom reported by the patient is considered one of the classic clinical manifestations of diabetes? A) Excessive thirst B) Gradual weight gain C) Overwhelming fatigue D) Recurrent blurred vision

A) Excessive thirst The classic symptoms of diabetes are polydipsia (excessive thirst), polyuria, (excessive urine output), and polyphagia (increased hunger). Weight gain, fatigue, and blurred vision may all occur with type 2 diabetes, but are NOT classic manifestations.

Laboratory results have been obtained for a 50-year-old patient with a 15-year history of type 2 diabetes. Which result reflects the expected pattern accompanying macrovascular disease as a complication of diabetes? A) Increased triglyceride levels B) Increased high-density lipoproteins (HDL) C) Decreased low-density lipoproteins (LDL) D) Decreased very-low-density lipoproteins (VLDL)

A) Increased triglyceride levels Macrovascular complications of diabetes include changes to large- and medium-sized blood vessels. They include cerebrovascular, cardiovascular, and peripheral vascular disease. Increased triglyceride levels are associated with these macrovascular changes. Increased HDL, decreased LDL, and decreased VLDL are positive in relation to atherosclerosis development.

A patient is admitted with diabetes mellitus, malnutrition, and cellulitis. The patient's potassium level is 5.6 mEq/L. The nurse understands that what could be contributing factors for this laboratory result (select all that apply)? A) The level may be increased as a result of dehydration that accompanies hyperglycemia. B) The patient may be excreting extra sodium and retaining potassium because of malnutrition. C) The level is consistent with renal insufficiency that can develop with renal nephropathy. D) The level may be raised as a result of metabolic ketoacidosis caused by hyperglycemia. Correct E) This level demonstrates adequate treatment of the cellulitis and effective serum glucose control.

A) The level may be increased as a result of dehydration that accompanies hyperglycemia. C) The level is consistent with renal insufficiency that can develop with renal nephropathy. E) This level demonstrates adequate treatment of the cellulitis and effective serum glucose control. The additional stress of cellulitis may lead to an increase in the patient's serum glucose levels. Dehydration may cause hemoconcentration, resulting in elevated serum readings. Kidneys may have difficulty excreting potassium if renal insufficiency exists. Finally, the nurse must consider the potential for metabolic ketoacidosis since potassium will leave the cell when hydrogen enters in an attempt to compensate for a low pH.

In which order will the nurse take these steps to prepare NPH 20 units and regular insulin 2 units using the same syringe? (Put a comma and a space between each answer choice [A, B, C, D, E]). a. Rotate NPH vial. b. Withdraw regular insulin. c. Withdraw 20 units of NPH. d. Inject 20 units of air into NPH vial. e. Inject 2 units of air into regular insulin vial

ANS: A, D, E, B, C When mixing regular insulin with NPH, it is important to avoid contact between the regular insulin and the additives in the NPH that slow the onset, peak, and duration of activity in the longer-acting insulin.

Which question during the assessment of a diabetic patient will help the nurse identify autonomic neuropathy? a. "Do you feel bloated after eating?" b. "Have you seen any skin changes?" c. "Do you need to increase your insulin dosage when you are stressed?" d. "Have you noticed any painful new ulcerations or sores on your feet?"

ANS: A Autonomic neuropathy can cause delayed gastric emptying, which results in a bloated feeling for the patient. The other questions are also appropriate to ask but would not help in identifying autonomic neuropathy

A 38-year-old patient who has type 1 diabetes plans to swim laps daily at 1:00 PM. The clinic nurse will plan to teach the patient to a. check glucose level before, during, and after swimming. b. delay eating the noon meal until after the swimming class. c. increase the morning dose of neutral protamine Hagedorn (NPH) insulin. d. time the morning insulin injection so that the peak occurs while swimming.

ANS: A The change in exercise will affect blood glucose, and the patient will need to monitor glucose carefully to determine the need for changes in diet and insulin administration. Because exercise tends to decrease blood glucose, patients are advised to eat before exercising. Increasing the morning NPH or timing the insulin to peak during exercise may lead to hypoglycemia, especially with the increased exercise.

A 55-year-old female patient with type 2 diabetes has a nursing diagnosis of imbalanced nutrition: more than body requirements. Which goal is most important for this patient? a. The patient will reach a glycosylated hemoglobin level of less than 7%. b. The patient will follow a diet and exercise plan that results in weight loss. c. The patient will choose a diet that distributes calories throughout the day. d. The patient will state the reasons for eliminating simple sugars in the diet.

ANS: A The complications of diabetes are related to elevated blood glucose, and the most important patient outcome is the reduction of glucose to near-normal levels. The other outcomes also are appropriate but are not as high in priority

Which patient action indicates good understanding of the nurse's teaching about administration of aspart (NovoLog) insulin? a. The patient avoids injecting the insulin into the upper abdominal area. b. The patient cleans the skin with soap and water before insulin administration. c. The patient stores the insulin in the freezer after administering the prescribed dose. d. The patient pushes the plunger down while removing the syringe from the injection site.

ANS: B Cleaning the skin with soap and water or with alcohol is acceptable. Insulin should not be frozen. The patient should leave the syringe in place for about 5 seconds after injection to be sure that all the insulin has been injected. The upper abdominal area is one of the preferred areas for insulin injection.

Which action should the nurse take after a 36-year-old patient treated with intramuscular glucagon for hypoglycemia regains consciousness? a. Assess the patient for symptoms of hyperglycemia. b. Give the patient a snack of peanut butter and crackers. c. Have the patient drink a glass of orange juice or nonfat milk. d. Administer a continuous infusion of 5% dextrose for 24 hours.

ANS: B Rebound hypoglycemia can occur after glucagon administration, but having a meal containing complex carbohydrates plus protein and fat will help prevent hypoglycemia. Orange juice and nonfat milk will elevate blood glucose rapidly, but the cheese and crackers will stabilize blood glucose. Administration of IV glucose might be used in patients who were unable to take in nutrition orally. The patient should be assessed for symptoms of hypoglycemia after glucagon administration

Which action should the nurse take after a 36-year-old patient treated with intramuscular glucagon for hypoglycemia regains consciousness? a. Assess the patient for symptoms of hyperglycemia. b. Give the patient a snack of peanut butter and crackers. c. Have the patient drink a glass of orange juice or nonfat milk. d. Administer a continuous infusion of 5% dextrose for 24 hours.

ANS: B Rebound hypoglycemia can occur after glucagon administration, but having a meal containing complex carbohydrates plus protein and fat will help prevent hypoglycemia. Orange juice and nonfat milk will elevate blood glucose rapidly, but the cheese and crackers will stabilize blood glucose. Administration of IV glucose might be used in patients who were unable to take in nutrition orally. The patient should be assessed for symptoms of hypoglycemia after glucagon administration.

A 26-year-old female with type 1 diabetes develops a sore throat and runny nose after caring for her sick toddler. The patient calls the clinic for advice about her symptoms and a blood glucose level of 210 mg/dL despite taking her usual glargine (Lantus) and lispro (Humalog) insulin. The nurse advises the patient to a. use only the lispro insulin until the symptoms are resolved. b. limit intake of calories until the glucose is less than 120 mg/dL. c. monitor blood glucose every 4 hours and notify the clinic if it continues to rise. d. decrease intake of carbohydrates until glycosylated hemoglobin is less than 7%.

ANS: C Infection and other stressors increase blood glucose levels and the patient will need to test blood glucose frequently, treat elevations appropriately with lispro insulin, and call the health care provider if glucose levels continue to be elevated. Discontinuing the glargine will contribute to hyperglycemia and may lead to diabetic ketoacidosis (DKA). Decreasing carbohydrate or caloric intake is not appropriate because the patient will need more calories when ill. Glycosylated hemoglobin testing is not used to evaluate short-term alterations in blood glucose.

A 26-year-old patient with diabetes rides a bicycle to and from work every day. Which site should the nurse teach the patient to administer the morning insulin? a. thigh. b. buttock. c. abdomen. d. upper arm.

ANS: C Patients should be taught not to administer insulin into a site that will be exercised because exercise will increase the rate of absorption. The thigh, buttock, and arm are all exercised by riding a bicycle.

A 48-year-old male patient screened for diabetes at a clinic has a fasting plasma glucose level of 120 mg/dL (6.7 mmol/L). The nurse will plan to teach the patient about a. self-monitoring of blood glucose. b. using low doses of regular insulin. c. lifestyle changes to lower blood glucose. d. effects of oral hypoglycemic medications.

ANS: C The patient's impaired fasting glucose indicates prediabetes, and the patient should be counseled about lifestyle changes to prevent the development of type 2 diabetes. The patient with prediabetes does not require insulin or oral hypoglycemics for glucose control and does not need to self-monitor blood glucose.

The nurse is assessing a 22-year-old patient experiencing the onset of symptoms of type 1 diabetes. Which question is most appropriate for the nurse to ask? a. "Are you anorexic?" b. "Is your urine dark colored?" c. "Have you lost weight lately?" d. "Do you crave sugary drinks?"

ANS: C Weight loss occurs because the body is no longer able to absorb glucose and starts to break down protein and fat for energy. The patient is thirsty but does not necessarily crave sugar-containing fluids. Increased appetite is a classic symptom of type 1 diabetes. With the classic symptom of polyuria, urine will be very dilute

Which finding indicates a need to contact the health care provider before the nurse administers metformin (Glucophage)? a. The patient's blood glucose level is 174 mg/dL. b. The patient has gained 2 lb (0.9 kg) since yesterday. c. The patient is scheduled for a chest x-ray in an hour. d. The patient's blood urea nitrogen (BUN) level is 52 mg/dL.

ANS: D The BUN indicates possible renal failure, and metformin should not be used in patients with renal failure. The other findings are not contraindications to the use of metformin.

A 28-year-old male patient with type 1 diabetes reports how he manages his exercise and glucose control. Which behavior indicates that the nurse should implement additional teaching? a. The patient always carries hard candies when engaging in exercise. b. The patient goes for a vigorous walk when his glucose is 200 mg/dL. c. The patient has a peanut butter sandwich before going for a bicycle ride. d. The patient increases daily exercise when ketones are present in the urine.

ANS: D When the patient is ketotic, exercise may result in an increase in blood glucose level. Type 1 diabetic patients should be taught to avoid exercise when ketosis is present. The other statements are correct

A newly diagnosed type 1 diabetic patient likes to run 3 miles several mornings a week. Which teaching will the nurse implement about exercise for this patient? a. "You should not take the morning NPH insulin before you run." b. "Plan to eat breakfast about an hour before your run." c. "Afternoon running is less likely to cause hypoglycemia." d. "You may want to run a little farther if your glucose is very high."

B Rationale: Blood sugar increases after meals, so this will be the best time to exercise. NPH insulin will not peak until mid-afternoon and is safe to take before a morning run. Running can be done in either the morning or afternoon. If the glucose is very elevated, the patient should postpone the run. Cognitive Level: Application Text Reference: p. 1269 Nursing Process: Implementation NCLEX: Physiological Integrity

A 1200-calorie diet and exercise are prescribed for a patient with newly diagnosed type 2 diabetes. The patient tells the nurse, "I hate to exercise! Can't I just follow the diet to keep my glucose under control?" The nurse teaches the patient that the major purpose of exercise for diabetics is to a. increase energy and sense of well-being, which will help with body image. b. facilitate weight loss, which will decrease peripheral insulin resistance. c. improve cardiovascular endurance, which is important for diabetics. d. set a successful pattern, which will help in making other needed changes.

B Rationale: Exercise is essential to decrease insulin resistance and improve blood glucose control. Increased energy, improved cardiovascular endurance, and setting a pattern of success are secondary benefits of exercise, but they are not the major reason. Cognitive Level: Application Text Reference: p. 1269 Nursing Process: Implementation NCLEX: Physiological Integrity

A patient with type 1 diabetes has an unusually high morning glucose measurement, and the health care provider wants the patient evaluated for possible Somogyi effect. The nurse will plan to a. administer an increased dose of NPH insulin in the evening. b. obtain the patient's blood glucose at 3:00 in the morning. c. withhold the nighttime snack and check the glucose at 6:00 AM. d. check the patient for symptoms of hypoglycemia at 2:00 to 4:00 AM.

B Rationale: In the Somogyi effect, the patient's blood glucose drops in the early morning hours (in response to excess insulin administration), which causes the release of hormones that result in a rebound hyperglycemia. It is important to check the blood glucose in the early morning hours to detect the initial hypoglycemia. An increased evening NPH dose or holding the nighttime snack will further increase the risk for early morning hypoglycemia. Information about symptoms of hypoglycemia will not be as accurate as checking the patient's blood glucose in determining whether the patient has the Somogyi effect. Cognitive Level: Application Text Reference: pp. 1263-1264 Nursing Process: Planning NCLEX: Physiological Integrity

A patient with newly diagnosed type 2 diabetes mellitus asks the nurse what "type 2" means in relation to diabetes. The nurse explains to the patient that type 2 diabetes differs from type 1 diabetes primarily in that with type 2 diabetes a. the patient is totally dependent on an outside source of insulin. b. there is decreased insulin secretion and cellular resistance to insulin that is produced. c. the immune system destroys the pancreatic insulin-producing cells. d. the insulin precursor that is secreted by the pancreas is not activated by the liver.

B Rationale: In type 2 diabetes, the pancreas produces insulin, but the insulin is insufficient for the body's needs or the cells do not respond to the insulin appropriately. The other information describes the physiology of type 1 diabetes. Cognitive Level: Application Text Reference: p. 1255 Nursing Process: Implementation NCLEX: Physiological Integrity

A patient with type 1 diabetes who uses glargine (Lantus) and lispro (Humalog) insulin develops a sore throat, cough, and fever. When the patient calls the clinic to report the symptoms and a blood glucose level of 210 mg/dl, the nurse advises the patient to a. use only the lispro insulin until the symptoms of infection are resolved. b. monitor blood glucose every 4 hours and notify the clinic if it continues to rise. c. decrease intake of carbohydrates until glycosylated hemoglobin is less than 7%. d. limit intake to non-calorie-containing liquids until the glucose is within the usual range.

B Rationale: Infection and other stressors increase blood glucose levels and the patient will need to test blood glucose frequently, treat elevations appropriately with insulin, and call the health care provider if glucose levels continue to be elevated. Discontinuing the glargine will contribute to hyperglycemia and may lead to DKA. Decreasing carbohydrate or caloric intake is not appropriate as the patient will need more calories when ill. Glycosylated hemoglobins are not used to test for short-term alterations in blood glucose. Cognitive Level: Application Text Reference: p. 1272 Nursing Process: Implementation NCLEX: Health Promotion and Maintenance

During a diabetes screening program, a patient tells the nurse, "My mother died of complications of type 2 diabetes. Can I inherit diabetes?" The nurse explains that a. as long as the patient maintains normal weight and exercises, type 2 diabetes can be prevented. b. the patient is at a higher than normal risk for type 2 diabetes and should have periodic blood glucose level testing. c. there is a greater risk for children developing type 2 diabetes when the father has type 2 diabetes. d. although there is a tendency for children of people with type 2 diabetes to develop diabetes, the risk is higher for those with type 1 diabetes.

B Rationale: Offspring of people with type 2 diabetes are at higher risk for developing type 2 diabetes. The risk can be decreased, but not prevented, by maintenance of normal weight and exercising. The risk for children of a person with type 1 diabetes to develop diabetes is higher when it is the father who has the disease. Offspring of people with type 2 diabetes are more likely to develop diabetes than offspring of those with type 1 diabetes. Cognitive Level: Application Text Reference: p. 1256 Nursing Process: Implementation NCLEX: Physiological Integrity

The health care provider orders oral glucose tolerance testing for a patient seen in the clinic. Which information from the patient's health history is most important for the nurse to communicate to the health care provider? a. The patient had a viral illness 2 months ago. b. The patient uses oral contraceptives. c. The patient runs several days a week. d. The patient has a family history of diabetes.

B Rationale: Oral contraceptive use may falsely elevate oral glucose tolerance test (OGTT) values. A viral 2 months previously illness may be associated with the onset of type 1 diabetes but will not falsely impact the OGTT. Exercise and a family history of diabetes both can affect blood glucose but will not lead to misleading information from the OGTT. Cognitive Level: Application Text Reference: p. 1267 Nursing Process: Assessment NCLEX: Physiological Integrity

A patient receives a daily injection of 70/30 NPH/regular insulin premix at 7:00 AM. The nurse expects that a hypoglycemic reaction is most likely to occur between a. 8:00 and 10:00 AM. b. 4:00 and 6:00 PM. c. 7:00 and 9:00 PM. d. 10:00 PM and 12:00 AM.

B Rationale: The greatest insulin effect with this combination occurs mid afternoon. The patient is not at a high risk at the other listed times, although hypoglycemia may occur. Cognitive Level: Comprehension Text Reference: p. 1260 Nursing Process: Evaluation NCLEX: Physiological Integrity

A patient with type 2 diabetes is scheduled for an outpatient coronary arteriogram. Which information obtained by the nurse when admitting the patient indicates a need for a change in the patient's regimen? a. The patient's most recent hemoglobin A1C was 6%. b. The patient takes metformin (Glucophage) every morning. c. The patient uses captopril (Capoten) for hypertension. d. The patient's admission blood glucose is 128 mg/dl.

B Rationale: To avoid lactic acidosis, metformin should not be used for 48 hours after IV contrast media are administered. The other patient data indicate that the patient is managing the diabetes appropriately. Cognitive Level: Application Text Reference: p. 1266 Nursing Process: Assessment NCLEX: Physiological Integrity

The nurse has taught a patient admitted with diabetes, cellulitis, and osteomyelitis about the principles of foot care. The nurse evaluates that the patient understands the principles of foot care if the patient makes what statement? A) "I should only walk barefoot in nice dry weather." B) "I should look at the condition of my feet every day." C) "I am lucky my shoes fit so nice and tight because they give me firm support." D) "When I am allowed up out of bed, I should check the shower water with my toes."

B) "I should look at the condition of my feet every day." Patients with diabetes mellitus need to inspect their feet daily for broken areas that are at risk for infection and delayed wound healing. Properly fitted (not tight) shoes should be worn at all times. Water temperature should be tested with the hands first.

A 54-year-old patient admitted with type 2 diabetes asks the nurse what "type 2" means. What is the most appropriate response by the nurse? A) "With type 2 diabetes, the body of the pancreas becomes inflamed." B) "With type 2 diabetes, insulin secretion is decreased, and insulin resistance is increased." C) "With type 2 diabetes, the patient is totally dependent on an outside source of insulin." D) "With type 2 diabetes, the body produces autoantibodies that destroy β-cells in the pancreas."

B) "With type 2 diabetes, insulin secretion is decreased, and insulin resistance is increased." In type 2 diabetes mellitus, the secretion of insulin by the pancreas is reduced, and/or the cells of the body become resistant to insulin. The pancreas becomes inflamed with pancreatitis. The patient is totally dependent on exogenous insulin and may have had autoantibodies destroy the β-cells in the pancreas with type 1 diabetes mellitus.

A type 1 diabetic patient who was admitted with severe hypoglycemia and treated tells the nurse, "I did not have any of the usual symptoms of hypoglycemia." Which question by the nurse will help identify a possible reason for the patient's hypoglycemic unawareness? a. "Do you use any calcium-channel blocking drugs for blood pressure?" b. "Have you observed any recent skin changes?" c. "Do you notice any bloating feeling after eating?" d. "Have you noticed any painful new ulcerations or sores on your feet?"

C Rationale: Hypoglycemic unawareness is caused by autonomic neuropathy, which would also cause delayed gastric emptying. Calcium-channel blockers are not associated with hypoglycemic unawareness, although -adrenergic blockers can prevent patients from having symptoms of hypoglycemia. Skin changes can occur with diabetes, but these are not associated with autonomic neuropathy. If the patient can feel painful areas on the feet, neuropathy has not occurred. Cognitive Level: Application Text Reference: p. 1281 Nursing Process: Assessment NCLEX: Physiological Integrity

During a clinic visit 3 months following a diagnosis of type 2 diabetes, the patient reports following a reduced-calorie diet. The patient has not lost any weight and did not bring the glucose-monitoring record. The nurse will plan to obtain a(n) a. fasting blood glucose level. b. urine dipstick for glucose. c. glycosylated hemoglobin level. d. oral glucose tolerance test.

C Rationale: The glycosylated hemoglobin (Hb A1C) test shows the overall control of glucose over 90 to 120 days. A fasting blood level indicates only the glucose level at one time. Urine glucose testing is not an accurate reflection of blood glucose level and does not reflect the glucose over a prolonged time. Oral glucose tolerance testing is done to diagnose diabetes, but is not used for monitoring glucose control once diabetes has been diagnosed. Cognitive Level: Application Text Reference: pp. 1258-1259 Nursing Process: Planning NCLEX: Physiological Integrity

The patient received regular insulin 10 units subcutaneously at 8:30 PM for a blood glucose level of 253 mg/dL. The nurse plans to monitor this patient for signs of hypoglycemia at which time related to the insulin's peak action? A) 8:40 PM to 9:00 PM B) 9:00 PM to 11:30 PM C) 10:30 PM to 1:30 AM D) 12:30 AM to 8:30 AM

C) 10:30 PM to 1:30 AM Regular insulin exerts peak action in 2 to 5 hours, making the patient most at risk for hypoglycemia between 10:30 PM and 1:30 AM. Rapid-acting insulin's onset is between 10-30 minutes with peak action and hypoglycemia most likely to occur between 9:00 PM and 11:30 PM. With intermediate acting insulin, hypoglycemia may occur from 12:30 AM to 8:30 AM.

A 65-year-old patient with type 2 diabetes has a urinary tract infection (UTI). The unlicensed assistive personnel (UAP) reported to the nurse that the patient's blood glucose is 642 mg/dL and the patient is hard to arouse. When the nurse assesses the urine, there are no ketones present. What collaborative care should the nurse expect for this patient? A) Routine insulin therapy and exercise B) Administer a different antibiotic for the UTI. C) Cardiac monitoring to detect potassium changes D) Administer IV fluids rapidly to correct dehydration.

C) Cardiac monitoring to detect potassium changes This patient has manifestations of hyperosmolar hyperglycemic syndrome (HHS). Cardiac monitoring will be needed because of the changes in the potassium level related to fluid and insulin therapy and the osmotic diuresis from the elevated serum glucose level. Routine insulin would not be enough, and exercise could be dangerous for this patient. Extra insulin will be needed. The type of antibiotic will not affect HHS. There will be a large amount of IV fluid administered, but it will be given slowly because this patient is older and may have cardiac or renal compromise requiring hemodynamic monitoring to avoid fluid overload during fluid replacement.

A college student is newly diagnosed with type 1 diabetes. She now has a headache, changes in her vision, and is anxious, but does not have her portable blood glucose monitor with her. Which action should the campus nurse advise her to take? A) Eat a piece of pizza. B) Drink some diet pop. C) Eat 15 g of simple carbohydrates. D) Take an extra dose of rapid-acting insulin.

C) Eat 15 g of simple carbohydrates. When the patient with type 1 diabetes is unsure about the meaning of the symptoms she is experiencing, she should treat herself for hypoglycemia to prevent seizures and coma from occurring. She should also be advised to check her blood glucose as soon as possible. The fat in the pizza and the diet pop would not allow the blood glucose to increase to eliminate the symptoms. The extra dose of rapid-acting insulin would further decrease her blood glucose.

A patient, who is admitted with diabetes mellitus, has a glucose level of 380 mg/dL and a moderate level of ketones in the urine. As the nurse assesses for signs of ketoacidosis, which respiratory pattern would the nurse expect to find? A) Central apnea B) Hypoventilation C) Kussmaul respirations D) Cheyne-Stokes respirations

C) Kussmaul respirations In diabetic ketoacidosis, the lungs try to compensate for the acidosis by blowing off volatile acids and carbon dioxide. This leads to a pattern of Kussmaul respirations, which are deep and nonlabored.

The newly diagnosed patient with type 2 diabetes has been prescribed metformin (Glucophage). What should the nurse tell the patient to best explain how this medication works? A) Increases insulin production from the pancreas. B) Slows the absorption of carbohydrate in the small intestine. C) Reduces glucose production by the liver and enhances insulin sensitivity. D) Increases insulin release from the pancreas, inhibits glucagon secretion, and decreases gastric emptying.

C) Reduces glucose production by the liver and enhances insulin sensitivity. Metformin is a biguanide that reduces glucose production by the liver and enhances the tissue's insulin sensitivity. Sulfonylureas and meglitinides increase insulin production from the pancreas. α-glucosidase inhibitors slow the absorption of carbohydrate in the intestine. Glucagon-like peptide receptor agonists increase insulin synthesis and release from the pancreas, inhibit glucagon secretion, and decrease gastric emptying.

When comparing the pathophysiology of type 1 and type 2 diabetes, which statement would be correct for a patient with type 2 diabetes who was admitted to the hospital with pneumonia? A. the patient must receive insulin therapy to prevent the development of ketoacidosis B. the patient has islet cell antibodies that have destroyed the ability of the pancreas to produce insulin C. The patient has minimal or absent endogenous insulin secretion and requires daily insulin injections D. the patient may have sufficient endogenous insulin to prevent ketosis but is at risk for development of hyperosmolar hyperglycemic syndrome

D

A patient with type 1 diabetes has received diet instruction as part of the treatment plan. The nurse determines a need for additional instruction when the patient says, a. "I may have an occasional alcoholic drink if I include it in my meal plan." b. "I will need a bedtime snack because I take an evening dose of NPH insulin." c. "I will eat meals as scheduled, even if I am not hungry, to prevent hypoglycemia." d. "I may eat whatever I want, as long as I use enough insulin to cover the calories."

D Rationale: Most patients with type 1 diabetes need to plan diet choices very carefully. Patients who are using intensified insulin therapy have considerable flexibility in diet choices but still should restrict dietary intake of items such as fat, protein, and alcohol. The other patient statements are correct and indicate good understanding of the diet instruction. Cognitive Level: Application Text Reference: p. 1268 Nursing Process: Evaluation NCLEX: Physiological Integrity

The nurse teaches the diabetic patient who rides a bicycle to work every day to administer morning insulin into the a. thigh. b. buttock. c. arm. d. abdomen.

D Rationale: Patients should be taught not to administer insulin into a site that will be exercised because exercise will increase the rate of absorption. The thigh, buttock, and arm are all exercised by riding a bicycle. Cognitive Level: Application Text Reference: p. 1262 Nursing Process: Implementation NCLEX: Physiological Integrity

A college student who has type 1 diabetes normally walks each evening as part of an exercise regimen. The student now plans to take a swimming class every day at 1:00 PM. The clinic nurse teaches the patient to a. delay eating the noon meal until after the swimming class. b. increase the morning dose of neutral protamine Hagedorn (NPH) insulin on days of the swimming class. c. time the morning insulin injection so that the peak occurs while swimming. d. check glucose level before, during, and after swimming.

D Rationale: The change in exercise will affect blood glucose, and the patient will need to monitor glucose carefully to determine the need for changes in diet and insulin administration. Because exercise tends to decrease blood glucose, patients are advised to eat before exercising. Increasing the morning NPH or timing the insulin to peak during exercise may lead to hypoglycemia, especially with the increased exercise. Cognitive Level: Application Text Reference: p. 1269 Nursing Process: Implementation NCLEX: Health Promotion and Maintenance

A program of weight loss and exercise is recommended for a patient with impaired fasting glucose (IFG). When teaching the patient about the reason for these lifestyle changes, the nurse will tell the patient that a. the high insulin levels associated with this syndrome damage the lining of blood vessels, leading to vascular disease. b. although the fasting plasma glucose levels do not currently indicate diabetes, the glycosylated hemoglobin will be elevated. c. the liver is producing excessive glucose, which will eventually exhaust the ability of the pancreas to produce insulin, and exercise will normalize glucose production. d. the onset of diabetes and the associated cardiovascular risks can be delayed or prevented by weight loss and exercise.

D Rationale: The patient with IFG is at risk for developing type 2 diabetes, but this risk can be decreased with lifestyle changes. Glycosylated hemoglobin levels will not be elevated in IFG and the Hb A1C test is not included in prediabetes testing. Elevated insulin levels do not cause the damage to blood vessels that can occur with IFG. The liver does not produce increased levels of glucose in IFG. Cognitive Level: Application Text Reference: p. 1255 Nursing Process: Implementation NCLEX: Physiological Integrity

A patient screened for diabetes at a clinic has a fasting plasma glucose level of 120 mg/dl (6.7 mmol/L). The nurse will plan to teach the patient about a. use of low doses of regular insulin. b. self-monitoring of blood glucose. c. oral hypoglycemic medications. d. maintenance of a healthy weight.

D Rationale: The patient's impaired fasting glucose indicates prediabetes and the patient should be counseled about lifestyle changes to prevent the development of type 2 diabetes. The patient with prediabetes does not require insulin or the oral hypoglycemics for glucose control and does not need to self-monitor blood glucose. Cognitive Level: Application Text Reference: p. 1255 Nursing Process: Planning NCLEX: Physiological Integrity

The nurse caring for a patient hospitalized with diabetes mellitus would look for which laboratory test result to obtain information on the patient's past glucose control? A) Prealbumin level B) Urine ketone level C) Fasting glucose level D) Glycosylated hemoglobin level

D) Glycosylated hemoglobin level A glycosylated hemoglobin level detects the amount of glucose that is bound to red blood cells (RBCs). When circulating glucose levels are high, glucose attaches to the RBCs and remains there for the life of the blood cell, which is approximately 120 days. Thus the test can give an indication of glycemic control over approximately 2 to 3 months. The prealbumin level is used to establish nutritional status and is unrelated to past glucose control. The urine ketone level will only show that hyperglycemia or starvation is probably currently occurring. The fasting glucose level only indicates current glucose control.

You are caring for a patient with newly diagnosed type 1 diabetes. What information is essential to include in your patient teaching before discharge from the hospital (select all that apply)? a. Insulin administration b. Elimination of sugar from diet c. Need to reduce physical activity d. Hypoglycemia prevention, symptoms, and treatment

a. Insulin administration d. Hypoglycemia prevention, symptoms, and treatment

A 72-year-old woman is diagnosed with diabetes. The nurse recognizes that management of diabetes in the older adult a. does not require as tight glucose control as in younger diabetics. b. is usually not treated unless the patient becomes severely hyperglycemic. c. does not include treatment with insulin because of limited dexterity and vision. d. usually requires that a younger family member be responsible for care of the patient.

a. Rationale: Because the clinical manifestations of long-term complications of diabetes take 10 to 20 years to develop, and because tight glucose control in the older patient is associated with an increase frequency of hypoglycemia, the goals for glycemic control are not as rigid as in the younger population. Treatment is indicated, and insulin may be used if the patient does not respond to oral agents. The patient's needs, rather than age, determine the responsibility of other in care.

Two days following a self-managed hypoglycemic episode at home, the patient tells the nurse that his blood glucose levels since the episode have been between 80 and 90mg/dL. The best response by the nurse is, a. "That is a good range for your glucose levels." b. "You should call your health care provider because you need to have your insulin increased." c. "That level is too low in view of your recent hypoglycemia, and you should increase your food intake." d. "You should only take half your insulin dosage for the next few days to get your glucose level back to normal."

a. Rationale: Blood glucose levels of 80 to 90mg/dL (4.5-5 mmol/L) are within the normal range and are desired in the patient with diabetes, even following a recent hypoglycemic episode. Hypoglycemia is often caused by a single event, such as skipping a meal or taking too much insulin or vigorous exercise; once corrected, normal control should be maintained.

A patient with diabetes calls the clinic because she is experiencing nausea and flulike symptoms. The nurse advises the patient to a. administer the usual insulin dosage. b. hold fluid intake until the nausea subsides. c. come to the clinic immediately for evaluation and treatment. d. monitor the blood glucose every 1 to 2 hours and call if the glucose rises over 150 mg/dL (8.3 mmol/L)

a. Rationale: During minor illnesses, the patient with diabetes should continue drug therapy and food intake. Insulin is important because counter regulatory hormones may raise blood glucose during the stress of illness, and food or a carbohydrate liquid substitution is important because during illness the body requires extra energy to deal with the stress of the illness. Blood glucose monitoring should be done every 4 hour, and the health care provider should be notified if the level is >240mg/dL (13.9 mmol/L) or if fever, ketonuria, or nausea and vomiting occur.

The major advantage of using an insulin pump is that a. tight glycemic control can be maintained. b. errors in insulin dosing are less likely to happen. c. complications of insulin therapy are prevented. d. frequent blood glucose monitoring is unnecessary.

a. Rationale: Insulin pumps provide tight glycemic control by continuous subcutaneous insulin infusion based on the patient's basal profile, with bolus doses at mealtime at the patient's discretion. Errors in insulin dosing and complications of insulin therapy are still potential risks with insulin pumps.

When caring for a patient with metabolic syndrome, the nurse gives the highest priority to teaching the patient about a. maintaining a normal weight. b. performing daily aerobic exercise. c. eliminating red mean from the diet. d. monitoring the blood glucose periodically.

a. Rationale: Metabolic syndrome is a cluster of abnormalities that include elevated insulin levels, elevated triglycerides and low-density lipoproteins (LDL), and decreased high-density lipoproteins (HDL). These abnormalities greatly increase the risk for cardiovascular disease associated with diabetes that can be prevented or delayed with weight loss. Exercise is also important, but normal weight is most important.

A diabetic patient is learning to mix regular insulin and NPH insulin in the same syringe. The nurse determines that additional teaching is needed when the patient a. withdraws the NPH dose in the syringe first b. injects air equal to the NPH dose into the NPH vial first. c. removes any air bubbles after withdrawing the first insulin. d. adds air equal to the insulin dose into the regular vial and withdraws the dose.

a. Rationale: When mixing regular with a longer-acting insulin, regular insulin should always be drawn in the syringe first to prevent contamination of the regular insulin vial with longer-acting insulin additives. Air is added in the neutral protamine Hagedorn (NPH) vial; then air is added to the regular vial, and the regular insulin is withdrawn, bubbles are removed, and the dose of NPH is withdrawn.

Which are appropriate therapies for patients with diabetes mellitus (select all that apply)? a. Use of statins to treat dyslipidemia b. Use of diuretics to treat nephropathy c. Use of ACE inhibitors to treat nephropathy d. Use of laser photocoagulation to treat retinopathy

a. Use of statins to treat dylipidemia c. Use of ACE inhibitors to treat nephropathy d. Use of laser photocoagulation to treat retinopathy

The client diagnosed with type 1 diabetes is found lying unconscious on the floor of the bathroom. Which intervention should the nurse implement first? a. administer 50% dextrose IVP b. notify the HCP c. move the client to the ICU d. check the serum glucose level

a. administer 50% dextrose IVP dextrose is only given if the client is found unconscious and the nurse suspect hypoglycemia. This will arouse the patient immediately

The nurses adminstered 28 units of Humulin N, an intermediate-acting insulin, to a client diagnosed with type 1 diabetes at 1600. Which intervention should the nurse implement? a. ensure the client eats the bedtime snack b. determine how much food the client ate at lunch c. perform a glucometer reading at 0700 d. offer the client protein after administering insulin

a. ensure the client eats the bedtime snack

A 72 year old woman is diagnosed with diabetes. What does the nurse recognize about the management of diabetes in the older adult? a. it is more difficult to achieve strict glucose control than in younger patients b. it usually is not treated unless the patient becomes severely hyperglycemic c. it does not include treatment with insulin because of limited dexterity and vision d. it usually requires that a younger family member be responsible for care of the patient

a. it is more difficult to achieve strict glucose control than in younger patients

The patient with type 2 diabetes is being put on acarbose (precose) and wants to know why she is taking it. What should the nurse include in this patients teaching (select all that apply) a. take it with the first bite of each meal b. it is not used in patients with HF c. endogenous glucose production is decreased d. effectiveness is measured by 2 hour postprandial glucose e. it delays glucose absorption from the GI tract

a. take it with the first bite of each meal d. effectiveness is measured by 2 hour postprandial glucose e. it delays glucose absorption from the GI tract

the diabetic educator is teach a class on diabetes type 1 and is discussing sick-day rules. Which intervention should the diabetes educator include in the discussion? select all that apply a. take the diabetic medication even if unable to eat the clients normal diabetic diet b. if unable to eat, drink liquids equal to the clients normal caloric diet c. if is not necessary to notify the HCP if ketones are in the urine d. test blood glucose levels and test urine ketones once a day and keep a record e. call the HCP if glucose levels are higher than 180 mg/dL

a. take the diabetic medication even if unable to eat the clients normal diabetic diet b. if unable to eat, drink liquids equal to the clients normal caloric diet e. call the HCP if glucose levels are higher than 180 mg/dL

The nurse is developing a care plan for the client diagnosed with type 1 diabetes. The nurse identifies the problem "high risk for hyperglycemia related to noncompliance with the medication regimen." What statement is an appropriate short-term goal for the client a. the client will have a blood glucose level between 90 and 140 mg/dL b. the client will demonstrate appropriate insulin injection technique c. the nurse will monitor the clients blood glucose levels 4 time a day d. the client will maintain normal kidney function with 30 ml/hr urine output

a. the client will have a blood glucose level between 90 and 140 mg/dL

Which clinical manifestations may be seen in a client experiencing diabetic ketoacidosis (hyperglycemia)? (Select all that apply.) a. thirst b. polyuria c. bradycardia d. Kussmaul's sign e. dry mucous membranes f. fruity breath odor

a. thirst b. polyuria d. Kussmaul's sign e. dry mucous membranes f. fruity breath odor

when teaching the patient with Type 1 diabetes, what should the nurse emphasize as the major disadvantage of using an insulin pump? a. tight glycemic control can be maintained b. errors in insulin dosing are less likely to occur c. complications of insulin therapy are prevented d. frequent blood glucose monitoring is unnecessary

a. tight glycemic control can be maintained

A patient with diabetes is learning to mix regular insulin and NPH insulin in the same syringe. The nurse determines that additional teaching is needed when the patient does what? a. withdraws the NPH dose into the syringe first b. infects air equal to the NPH dose into the NPH vial first c. removes any air bubbles after withdrawing the first insulin d. adds air equal to the insulin dose into the regular vial and withdraws the dose

a. withdraws the NPH dose into the syringe first

Which are appropriate therapies for patients with diabetes mellitus (select all that apply)? a.Use of statins to treat dyslipidemia b.Use of diuretics to treat nephropathy c.Use of ACE inhibitors to treat nephropathy d.Use of serotonin agonists to decrease appetite e.Use of laser photocoagulation to treat retinopathy

a.Use of statins to treat dyslipidemia c.Use of ACE inhibitors to treat nephropathy e.Use of laser photocoagulation to treat retinopathy

During routine health screening, a patient is found to have a fasting plasma glucose (FPG) of 132 mg/dL (7.33. mmol/L). At a follow-up visit, a diagnostic of diabetes would be made based on (select all that apply) a. glucosuria of 3+. b. an A1C of 7.5%. c. a FPG of ≥126mg/dL (6.9 mmol/L). d. random blood glucose of 126 mg/dL (7.0 mmol/L). e. a 2-hour oral glucose tolerance test (OGTT) of 190 mg/dL (10.5 mmol/L).

b,c. Rationale: The patient has one prior test result that meets criteria for a diagnosis of diabetes, but on a subsequent day must again have results from on of the three tests that meet the criteria for diabetes diagnosis. These criteria include a fasting plasma glucose level of ≥126mg/dL (7.0 mmol/L), or A1C ≥6.5% or a 2 hour OGTT level ≥200mg/dL (11.1 mmol/L). Both the fasting plasma glucose (FPG) and A1C would confirm a diagnosis of diabetes in this patient.

A client is newly diagnosed with type 1 diabetes mellitus and requires daily insulin injections. Which instruction should the nurse include in the teaching of insulin administration? a. Teach the family members to administer glucagon by injection if the client has a hyperglycemic reaction. b. Instruct the client about the necessity for compliance with prescribed insulin therapy. c. Teach the client that hypoglycemic reactions more likely to occur at the onset of action time. d. Instruct the client in the care of insulin container and syringe handling.

b. Instruct the client about the necessity for compliance with prescribed insulin therapy.

What is the priority action for the nurse to take if the patient with type 2 diabetes complains of blurred vision and irritability? a. Call the physician. b. Administer insulin as ordered. c. Check the patients blood glucose level. d. Assess for other neurologic symptoms.

c. Check the patients blood glucose level.

A patient with type 1 diabetes uses 20U of 70/30 neutral protamine Hagedorn (NPH/regular) in the morning and at 6:00pm. When teaching the patient about this regimen, the nurse stresses that a. hypoglycemia is most likely to occur before the noon meal. b. a set meal pattern with a bedtime snack is necessary to prevent hypoglycemia. c. flexibility in food intake is possible because insulin is available 24 hours/day. d. pre-meal glucose checks are required to determine needed changes in daily dosing.

b. Rationale: A split-mixed dose of insulin requires that the patient adhere to a set meal pattern to provide glucose for the action of the insulins, and a bedtime snack is usually required when patients take a long-acting insulin late in the day to prevent nocturnal hypoglycemia. Hypoglycemia is most likely to occur with this dose late in the afternoon and during the night. When premixed formulas are used, flexible dosing based on glucose levels is not recommended.

The following interventions are planned for a diabetic patient. Which intervention can the nurse delegate to nursing assistive personnel (NAP)? a. Discuss complications of diabetes. b. Check that the bath water is not too hot. c. Check the patient's technique for drawing up insulin. d. Teach the patient to use the glucometer for in-home glucose monitoring.

b. Rationale: Checking the temperature of the bath water is part of assisting with activities of daily living (ADLs) and within the scope of care for the nursing assistive personnel (NAP). Discussion of complications, teaching, and assessing learning are appropriate for RNs.

To prevent hyperglycemia or hypoglycemia with exercise, the nurse teaches the patient using glucose-lowering agents that exercise should be undertaken a. only after a 10- to 15-g carbohydrate snack is eaten. b. about 1 hour after a eating, when blood glucose levels are rising. c. when glucose monitoring reveals that the blood glucose is in the normal range. d. when blood glucose levels are high because exercise always has a hypoglycemic effect.

b. Rationale: During exercise, a diabetic person needs both adequate glucose to prevent exercise-induced hypoglycemia and adequate insulin because conterregulatory hormones are produced during the stress of exercise and may cause hyperglycemia. Exercise after meals is best, but a 10- to 15-g carbohydrate snack may be taken if exercise is performed before meals or is prolonged. Blood glucose levels should be monitored before, during, and after exercise to determine the effect of exercise on the levels.

Goals of nutritional therapy for the patient with type 2 diabetes include maintenance of a. ideal body weight. b. normal serum glucose and lipid levels. c. a special diet using diabetic foods. d. five small meals per day with a bedtime snack.

b. Rationale: Maintenance of as near-normal blood glucose levels as possible and a achievement of optimal serum lipid levels with dietary modification are believed to be the most important factors in preventing both short- and long-term complications of diabetes. There is no specific "diabetic diet," and use of dietetic foods is not necessary for diabetes control. Most diabetics eat three meals a day, and some require a bedtime snack for control of nighttime hypoglycemia. A reasonable weight, which may or may not be an ideal body weight, is also a goal of nutritional therapy.

the client with type 2 diabetes controlled with biguanide oral diabetic medication is scheduled for a CT scan with contrast of the abdomen to evaluate pancreatic function. Which intervention should the nurse implement? a. provide a high-fat diet 24 hour prior to test b. hold the biguanide med for 48 hours prior to test c. obtain an informed consent form for the test d. administer pancreatic enzymes prior to the test

b. hold the biguanide med for 48 hours prior to test

Polydipsia and polyuria related to diabetes mellitus are primarily due to a.the release of ketones from cells during fat metabolism. b.fluid shifts resulting from the osmotic effect of hyperglycemia. c.damage to the kidneys from exposure to high levels of glucose. d.changes in RBCs resulting from attachment of excessive glucose to hemoglobin.

b.fluid shifts resulting from the osmotic effect of hyperglycemia.

What describes the primary difference in treatment of DKA and hyperosmolar hyperglycemia syndrome (HHS) a. DKA requires administration of bicarbonate to correct acidosis b. potassium replacement is not necessary in management of HHS c. HHS requires greater fluid replacement to correct the dehydration d. administration of glucose is withheld in HHS until the blood glucose reaches a normal level

c. HHS requires greater fluid replacement to correct the dehydration

The nurse assesses the diabetic patient's technique of self-monitoring of blood glucose (SMBG) 3 months after initial instruction. An error in the performance of SMBG noted by the nurse that requires intervention is a. doing the SMBG before and after exercising. b. puncturing the finger on the side of the finger pad. c. cleaning the puncture site with alcohol before the puncture. d. holding the hand down for a few minutes before the puncture.

c. Rationale: Cleaning the puncture site with alcohol is not necessary and may interfere with test results and lead to drying and splitting of the fingertips. Washing the hands with warm water is adequate cleaning and promotes blood flow to the fingers. Blood flow is also increase by holding the hand down. Puncture on the side of the finger pad are less painful. Self-monitored blood glucose (SMBG) should be performed before and after exercise.

The nurse determines that a patient with a 2-hour OGTT of 152mg/dL has a. diabetes. b. impaired fasting glucose. c. impaired glucose tolerance. d. elevated glycosylated hemoglobin (Hb)

c. Rationale: Impaired glucose tolerance exists when a 2-hour plasma glucose level is higher than normal but lower than the level diagnostic for diabetes (i.e., 140-199mg/dL). Impaired fasting glucose exists when fasting glucose levels are greater than the normal of 100mg/dL but less than the 126mg/dL diagnostic of diabetes. Both conditions represent a condition known as pre diabetes.

The nurse is assessing a newly admitted diabetic patient. Which of these observations should be addressed as a priority by the nurse? a. Bilateral numbness of both hands b. Stage II pressure ulcer on the right heel c. Rapid respirations with deep inspiration d. Areas of lumps and dents on the abdomen.

c. Rationale: Rapid deep respirations are symptoms of diabetic ketoacidosis (DKA). Stage II ulcers and bilateral numbness are chronic complications of diabetes. The lumps and dents on the abdomen indicate a need to teach the patient about site rotation.

A patient taking insulin has recorded fasting glucose levels above 200mg/dL (11.1 mmol/L) on awakening for the last five mornings. The nurse advises the patient to a. increase the evening insulin dose to prevent the dawn phenomenon. b. use a single dose insulin regimen with an intermediate-acting insulin. c. monitor the glucose level at bedtime, between 2:00 and 4:00 am, and on arising. d. decrease the evening insulin dosage to prevent night hypoglycemia and the Somogyi effect.

c. Rationale: The patient's elevated glucose on arising may be the result of either dawn phenomenon or Somogyi effect, and the best way to determine whether the patient needs more or less insulin is by monitoring the glucose at bedtime, between 2:00 and 4:00 am, and on arising. If predawn levels are below 60mg/dL, the insulin dose should be reduced, but if the 2:00 to 4:00 am blood glucose is high, the insulin should be increased.

When teaching the patient with diabetes about insulin administration, the nurse instructs the patient to a. pull back on the plunger after inserting the needle to check for blood. b. clean the skin at the injection site with an alcohol swab before each injection. c. consistently use the same size of the appropriate strength insulin syringe to avoid dosing errors. d. rotate injection sites from arms to thighs to abdomen with each injection to prevent lipodystrophies.

c. Rationale: U100 insulin must be used with a U100 syringe, but for those using low doses of insulin, syringes are available that have increments of 1 unit instead of 2 units. Errors can be made in dosing if patients switch back and forth between different sizes of syringes. Aspiration before injection of the insulin is not recommended, nor is the use of alcohol to clean the skin. Because the rate of peak serum concentrations varies with the site selected for injection, injections should be rotated within a particular area, such as the abdomen.

Ketoacidosis occurs as a complication of diabetes when a. illnesses causing nausea and vomiting lead to bicarbonate loss with body fluids. b. the glucose level becomes so high that osmotic diuresis promotes fluid and electrolyte loss. c. an insulin deficit causes the body to metabolize large amounts of fatty acids rather than glucose for energy. d. the patient skips meals after taking insulin, leading to rapid metabolism of glucose and breakdown of fats for energy.

c. Rationale: When insulin is insufficient and glucose cannot be used for cellular energy, the body releasees and breaks down stored fats and protein to meet energy needs. Free fatty acids from stored triglycerides are released and metabolized in the liver in such large quantities that ketones are formed. Ketones are acidic and alter the pH of the blood, causing acidosis. Osmotic diuresis occurs as a result of loss of both glucose and ketones in the urine.

Which patient should the nurse plan to teach how to prevent or delay the development of diabetes? a. an obese 50 year old hispanic woman b. a child whose father has type 1 diabetes c. a 34 year old woman whose parents both have type 2 diabetes d. a 12 year old boy whose father has maturity onset diabetes of the young (MODY)

c. a 34 year old woman whose parents both have type 2 diabetes

The nurse should observe the patient for symptoms of ketoacidosis when a. illness causing nausea and vomiting lead to bicarbonate loss with body fluids b. glucose levels become so high that osmotic diuresis promotes fluid and electrolyte loss c. an insulin deficit causes the body to metabolize large amounts of fatty acids rather than glucose for energy d. the patient skips meals after taking insulin, leading to rapid metabolism of glucose and breakdown of fats for energy

c. an insulin deficit causes the body to metabolize large amounts of fatty acids rather than glucose for energy

The client diagnosed with type 2 diabetes is admitted to the intensive care unit with hyperosmolar hyperglycemia nonketonic syndrome (HHNS) coma. Which assessment data should the nurse expect the client to exhibit? a. Kussmauls respiration b. kiarrhea and epigastric pain c. dry mucous membranes d. ketones breath odor

c. dry mucous membranes

The client received 10 units of Humulin R, a fast actign insulin, at 0700. At 1030 the UAP tells the nurse the client has a headache and is really acting "funny." Which intervention should the nurse implement first? a. Instruct the UAP to obtain the blood glucose level b. Have the client drink 8 oz of orange juice c. go to the clients room and assess the client for hypoglycemia d. prepare to administer 1 ampule 50% dextrose IV

c. go to the clients room and assess the client for hypoglycemia

the nurse is discussing ways to prevent DKA with the client diagnosed with type 1 diabetes. Which instruction is the most import to discuss with the client? a. refer the client to the American Diabetes Association b. do not take an OTC meds c. take the prescribed insulin even when unable to eat because of illness d. explain the need to get the annual flu and pneumonia vaccines

c. take the prescribed insulin even when unable to eat because of illness

A diabetic patient is found unconscious at home, and a family member calls the clinic. After determining that no glucometer is available, the nurse advise the family member to a. try to around the patient to drink some orange juice. b administer 10 U or regular insulin subcutaneously. c. call for an ambulance to transport the patient to a medical facility. d. administer glucagon 1mg intramuscularly (IM) or subcutaneously.

d. Rationale: If a diabetic patient is unconscious, immediate treatment for hypoglycemia must be given to prevent brain damage, and IM or subcutaneous administration of 1mg of glucagon should be done. If that unconsciousness has another cause, such as ketosis, the rise in glucose cause by the glucagon is not as dangerous as the low glucose level. Following administration of the glucagon, the patient should be transported to a medical facility for further treatment and evaluation. Insulin is contraindicated without knowledge of the patient's glucose level, and oral carbohydrate cannot be given when patients are unconscious.

The home care nurse should intervene to correct a patient whose insulin administration includes a. warming a pre filled refrigerated syringe in the hands before administration. b. storing syringes pre filled with NPH and regular insulin needle-up in the refrigerator. c. placing the insulin bottle currently in use in a small container on the bathroom countertop. d. mixing an evening dose of regular insulin with insulin glargine in one syringe for administration.

d. Rationale: Insulin glargine (Lantus), a long-acting insulin that is continuously released with no peak of action, cannot be diluted or mixed with any other insulin or solution. Mixed insulins should be stored needle-up in the refrigerator and warmed before administration. Currently used bottles of insulin can be kept at room temperature.

In addition to promoting the transport of glucose from the blood into the cell, insulin also a. enhances the breakdown of adipose tissue for energy. b. stimulates hepatic glycogenolysis and gluconeogenesis. c. prevents the transport of triglycerides into adipose tissue. d. accelerates the transport of amino acids into cells and their synthesis into protein.

d. Rationale: Insulin is an anabolic hormone, responsible for growth, repair, and storage, and it facilitates movement of amino acids into cells, synthesis of protein, storage of glucose as glycogen, and deposition of triglycerides and lipids as fat into adipose tissue. Glucagon is responsible for hepatic glycogenolysis and gluconeogenesis, and fat is used for energy when glucose levels are depleted.

Which of the following patients would a nurse plan to teach how to prevent or delay the development of diabetes? a. A 62 year-old obese white woman. b. An obese 50 year-old Hispanic woman. c. A child whose father has type 1 diabetes. d. A 34 year-old woman whose parents both have type 2 diabetes.

d. Rationale: Type 2 diabetes has a strong genetic influence, and offspring of parents who both have type 2 diabetes have an increased chance of developing it. Whereas type 1 diabetes is associated with genetic susceptibility related to human leukocyte antigens (HLAs), offspring of parents who both have type 1 diabetes have only a 6% to 10% chance of developing the disease. Lower risk factors for type 2 diabetes include obesity; being a Native American, Hispanic, or African-American; and being 55 years or older.

When comparing the pathophysiology of type 1 and type 2 diabetes, which statement would be correct for a patient with type 2 diabetes who was admitted to the hospital with pneumonia? a. The patient must receive insulin therapy to prevent the development of ketoacidosis. b. The patient has islet cell antibodies that have destroyed the ability of the pancreas to produce insulin. c. The patient has minimal or absent endogenous insulin secretion and requires daily insulin injections. d. The patient may have sufficient endogenous insulin secretion to prevent ketosis but is at risk for development of hyperosmolar hyperglycemic syndrome.

d. The patient may have sufficient endogenous insulin secretion to prevent ketosis but is at risk for development of hyperosmolar hyperglycemic syndrome.

following the teaching of foot care to a diabetic patient, the nurse determines that additional instruction is needed when the patients make which statement? a. I should wash my feet daily with soap and warm water b. i should always wear shoes to protect my feet from injury c. if my feet are cold, i should wear socks instead of using a heating pad d. ill know if i have sores or lesions on my feet because it will be painful

d. ill know if i have sores or lesions on my feet because it will be painful

The client diagnosed with HHNS was admitted yesterday with a blood glucose level of 780 mg/dL. The clients blood glucose level is now 300 mg/dL. Which intervention should the nurse implement? a. increase the regular insulin IV drip b. check the patients urine for ketones c. provide the client with a therapeutic diabetic meal d. notify the HCP to obtain an order to decrease insulin

d. notify the HCP to obtain an order to decrease insulin

which arterial blood gas results should.d the nurse expect in the client diagnosed with DKA a. pH 7.34, PaO2 99, PaCO2 48, HCO3 24 b. pH 7.38, PaO2 95, PaCO2 40, HCO3 22 c. pH 7.46, PaO2 85, PaCO2 30, HCO3 18 d. pH 7.30, PaO2 90, PaCO2 30, HCO3 18

d. pH 7.30, PaO2 90, PaCO2 30, HCO3 18


Conjuntos de estudio relacionados

Test 8 - Hazardous Material Safety

View Set

Chapter 2 Sentence and paragraph order

View Set

Chapter 3: The Internal Organization: Resources, Capabilities, Core Competencies, and Competitive Advantages

View Set

Retirement Planning Review Questions

View Set

Unit 3: Geometry in Engineering, Architecture, and Fine Arts, Part 2

View Set